Đến nội dung

Nguyenhuyen_AG nội dung

Có 785 mục bởi Nguyenhuyen_AG (Tìm giới hạn từ 29-03-2020)



Sắp theo                Sắp xếp  

#700708 $\frac{a^{2}}{b}+\frac{b^...

Đã gửi bởi Nguyenhuyen_AG on 23-01-2018 - 13:34 trong Bất đẳng thức và cực trị

Cho $a,b,c>0$. CMR:

$b) \frac{a^{2}}{b}+\frac{b^{2}}{c}+\frac{c^2}{a}\geq a+b+c+\frac{4(a-c)^2}{a+b+c}$ 

 

Ta có

\[\text{VT - VP} = \frac{a(b^2-ca)^2+c(a^2-2ab+bc)^2+b(ab-2ca+c^2)^2}{abc(a+b+c)} \geqslant 0.\]




#694385 Cực Trị Nâng cao

Đã gửi bởi Nguyenhuyen_AG on 08-10-2017 - 20:01 trong Bất đẳng thức và cực trị

Tìm giá trị nhỏ nhất của E = x + y biết x2 + y2 = 50 và 1 $\leq$ x $\leq$ 7; 1 $\leq$ y $\leq$ 7

 

Ta có

\[x+y = \sqrt{16 + \frac{2[(7x+27)(x-1)+(20x+7y)(y-1)+y(7-x)]}{19}} \geqslant  \sqrt{16} = 8.\]




#694236 $\sum {{x^3}} + 2\sum {{x^2...

Đã gửi bởi Nguyenhuyen_AG on 05-10-2017 - 20:48 trong Bất đẳng thức - Cực trị

Cho $x,y,z \ge 0$. Chứng minh :${x^3} + {y^3} + {z^3} + 2({x^2}y + {y^2}z + {z^2}x) \ge 3(x{y^2} + y{z^2} + z{x^2})$

 

Lời giải: https://diendantoanh...ào/#entry474480




#694234 $2x^{2} +3y^{2} + 4z^{2}$

Đã gửi bởi Nguyenhuyen_AG on 05-10-2017 - 20:39 trong Bất đẳng thức và cực trị

cho x,y,z>0 : xy+yz+xz=3. tìm min

A= $2x^{2} +3y^{2} + 4z^{2}$

 

Bài đẹp nhưng kết quả không đẹp. Đặt

\[P = 2x^{2} +3y^{2} + 4z^{2} - k(xy+yz+zx),\]

khi đó

\[P=\frac18\left( ky+kz-4\,x \right) ^{2}+{\frac{( {k}^{2}y+{k}^{2}z+4kz-24y) ^{2}}{24-{k}^{2}}}+{\frac {{z}^{2}( {k}^{3}+9{k}^{2}-96) }{24-{k}^{2}}}.\]

Như vậy nếu chọn $k$ sao cho ${k}^{3}+9{k}^{2}-96=0,k^2<24$ thì $P \geqslant 0$ tức $P$ có giá trị nhỏ nhất là $3k.$




#693766 $\frac{a+b}{c^2}+\frac{b+c}...

Đã gửi bởi Nguyenhuyen_AG on 26-09-2017 - 22:25 trong Bất đẳng thức - Cực trị

Cho $a,b,c$ là độ dài ba cạnh tam giác thỏa mãn: $abc=1$. Chứng minh rằng: $\frac{a+b}{c^2}+\frac{b+c}{a^2}+\frac{c+a}{b^2}\ge \frac{5}{2}(a^2+b^2+c^2)-\frac{1}{2}(ab+bc+ca)$

 

Viết bất đẳng thức lại dưới dạng thuần nhất

\[f(a,b,c) = \sum \frac{a+b}{c^2} - \frac{5(a^2+b^2+c^2) -ab-bc-ca}{2abc} \geqslant 0.\]

Giả sử $x,y,z$ là ba số thực dương, áp dụng phép thế Ravi ta có

\[f(a,b,c) = f(x+y,y+z,z+x) \equiv f(x,y,z),\]

\[f(x,y,z)= \frac{1}{2(x+y)^2(y+z)^2(z+x)^2} \sum (x^3+3x^2y+5xy^2+y^3+2x(x+y-z)^2)(x-y)^2 \geqslant 0.\]




#693604 $\frac{x}{2}+\frac{8x^{3}}{(x-2)(x+2)^{2}} >9$

Đã gửi bởi Nguyenhuyen_AG on 23-09-2017 - 22:02 trong Bất đẳng thức và cực trị

Bạn lấy x=9 hoặc bất kỳ đi 

VT ko bằng VP

 

Mình vẫn thấy nó đúng với $x=9$. :(




#693600 $\frac{x}{2}+\frac{8x^{3}}{(x-2)(x+2)^{2}} >9$

Đã gửi bởi Nguyenhuyen_AG on 23-09-2017 - 21:38 trong Bất đẳng thức và cực trị

bạn phân tích sai rồi

 

Mình kiểm tra thấy đúng mà nhỉ. :(




#693593 Bất đẳng thức AM-GM

Đã gửi bởi Nguyenhuyen_AG on 23-09-2017 - 21:15 trong Bất đẳng thức và cực trị

Bài 1: Cho các số thực a,b,c thỏa ab+7bc+ca=188. Tìm GTNN $P= 5a^{2}+11b^{2}+5c^{2}$

Bài 2: Cho a,b,c>0, a+b+c=3. Tìm GTNN P = a2+b2+c3

 

Bài 1. Ta có

\[5a^2+11b^2+5c^2-2(ab+7bc+ca) = \frac15(5a-b-c)^2+\frac65(3b-2c)^2 \geqslant 0.\]

Bài 2. Đặt $z = \frac{\sqrt{37}-1}{6} > 0,$ theo bất đẳng thức AM-GM ta có $c^3 \geqslant \frac{3z}{2}c^2-\frac{z^3}{2},$ và

\[a^2+b^2+\frac{3z}{2}c^2 - \frac{27z}{2(3z+1)} \cdot \frac{(a+b+c)^2}{9} = \frac{(3az-3bz-3cz+2a)^2+(3z+1)(2b-3cz)^2}{(3z+2)(6z+2)} \geqslant 0.\]

Do đó

\[P \geqslant a^2+b^2+\frac{3z}{2}c^2-\frac{z^3}{2} \geqslant \frac{27z}{2(3z+1)} - \frac{z^3}{2} = \frac{541-\sqrt{37^3}}{108}.\]

Đẳng thức xảy ra khi $a=b=\frac{19-\sqrt{37}}{12},c=\frac{\sqrt{37}-1}{6}.$




#693590 $\frac{x}{2}+\frac{8x^{3}}{(x-2)(x+2)^{2}} >9$

Đã gửi bởi Nguyenhuyen_AG on 23-09-2017 - 20:38 trong Bất đẳng thức và cực trị

Cho x>2. CMR:

$\frac{x}{2}+\frac{8x^{3}}{(x-2)(x+2)^{2}} >9$

 

Bởi vì

\[\frac{x}{2}+\frac{8x^{3}}{(x-2)(x+2)^{2}} -9 = \frac{(x^2+8x+8)(x-4)^2+16}{2(x-2)(x+2)^2} > 0.\]




#693209 Cho $a^2+b^2<1$. Chứng minh phương trình sau luôn có nghiệm

Đã gửi bởi Nguyenhuyen_AG on 17-09-2017 - 15:08 trong Bất đẳng thức và cực trị

Cho $a^2+b^2<1$. Chứng minh phương trình sau luôn có nghiệm

$(a^2+b^2-1)x^2 -2(ac+bd-1)x+c^2+d^2-1=0$

 

Phương trình này luôn có nghiệm $x$ bởi vì biệt thức

\[\Delta^{'}_x = \frac{(abd-b^2c-a+c)^2+(b-d)^2(1-a^2-b^2)}{1-b^2} \geqslant 0.\]




#693203 tìm max :xy+yz+zx

Đã gửi bởi Nguyenhuyen_AG on 17-09-2017 - 13:54 trong Bất đẳng thức và cực trị

Cho x,y,z thỏa mãn :$x^{2}+y^{2}+z^{2}=1-\frac{9}{16}xy$

Tìm MAX : P=xy+yz+zx

 

Xét \[P=\frac{8+12\sqrt5}{41}\left(x^{2}+y^{2}+z^{2}+\frac{9}{16}xy\right) - (xy+yz+zx),\] thì
\[P = {\frac { \left( 3 \sqrt{5}+2 \right)  \left( 12 \sqrt{5}y+12 \sqrt{5}z-32x-17y-8z \right) ^{2}}{10496}}+{\frac { \left( 21 \sqrt{5}+150 \right)  \left( 12 \sqrt{5}z-41y+8z \right) ^{2}}{430336}} \geqslant 0.\]

Do đó

\[xy+yz+zx \leqslant \frac{8+12\sqrt5}{41}\left(x^{2}+y^{2}+z^{2}+\frac{9}{16}xy\right).\]




#692250 $\sum \frac{a(3a^2+5bc)}{(b+c)^2}\geq...

Đã gửi bởi Nguyenhuyen_AG on 03-09-2017 - 18:57 trong Bất đẳng thức - Cực trị

c. $\sum \frac{a^3+abc}{b^3+c^3+abc}\geq 2$

 

Nếu có một số bằng $0$ thì bất đẳng thức hiển nhiên. Xét trường hợp các số đều dương, áp dụng bất đẳng thức Cauchy-Schwarz ta có

\[\sum \frac{a^3+abc}{b^3+c^3+abc} \geqslant \frac{\displaystyle \left[\sum(a^3+abc)\right]^2}{\displaystyle \sum (b^3+c^3+abc)(a^3+abc)}.\]

\[\left[\sum(a^3+abc)\right]^2 -2 \sum (b^3+c^3+abc)(a^3+abc) = \sum (a^2+bc) \sum a^2(a-b)(a-c) \ge 0.\]

Ta có điều phải chứng minh.




#692161 MAX $\frac{3a+2b+c}{(a+b)(b+c)(c+a)}$

Đã gửi bởi Nguyenhuyen_AG on 02-09-2017 - 22:25 trong Bất đẳng thức - Cực trị

$\left\{\begin{matrix} a,b,c>0 & \\ 3bc+4ca+5ab\leq 6abc & \end{matrix}\right.$

 

Tìm MAX:

 

$\frac{3a+2b+c}{(a+b)(b+c)(c+a)}$

 

Giá trị lớn nhất là $\frac3{16}$ đạt được khi 3 biến đều bằng nhau.




#691818 $\sqrt[3]{(a^2+1)(b^2+1)(c^2+1)}\leq (\frac...

Đã gửi bởi Nguyenhuyen_AG on 29-08-2017 - 20:47 trong Bất đẳng thức - Cực trị

Chỗ y đấy phải là t chứ ạ.

Em đóng góp thế thôi,chứ cách giải của anh cũng rất hay.

 

À đúng rồi, chỗ đấy anh gõ nhầm. Cám ơn em nhé.




#691607 $\sum \frac{x^2}{x^2+xy+xz} \leq...

Đã gửi bởi Nguyenhuyen_AG on 26-08-2017 - 20:11 trong Bất đẳng thức - Cực trị

$\sum \frac{x^2}{x^2+xy+xz} \leq \frac{\sum x^2}{\sum xy}$
với $x,y,z>0$

 

Ta có

\[\text{VT-VP} = \frac{x^2+y^2+z^2-xy-yz-zx}{xy+yz+zx} \geqslant 0.\]




#691604 $\sum {\frac{{a - b}}{b}...

Đã gửi bởi Nguyenhuyen_AG on 26-08-2017 - 20:09 trong Bất đẳng thức - Cực trị

Cho 3 số dương a,b,c : CMR $\sum {\frac{{a - b}}{b}}  \ge \frac{{{{(a - c)}^2}}}{{(a + b)(b + c)}}$

 

Ta có

\[\text{VT - VP}=\frac{c^2(a+b)(a-b)^2+b^2c(a-c)^2+ab(a+b)(b-c)^2}{abc(a+b)(b+c)} \geqslant 0.\]




#691602 $\sqrt[3]{(a^2+1)(b^2+1)(c^2+1)}\leq (\frac...

Đã gửi bởi Nguyenhuyen_AG on 26-08-2017 - 20:05 trong Bất đẳng thức - Cực trị

Cho các các số thực a,b,c thỏa mãn $min(xy,yz,zx)\geq 1$.

Chứng minh rằng:$\sqrt[3]{(x^2+1)(y^2+1)(z^2+1)}\leq (\frac{x+y+z}{3})^2 +1$

 

Giả sử $x \geqslant y \geqslant z$ và đặt $t=\frac{y+z}2$ thì $x\geqslant t \geqslant 1.$ Xét $f(x,y,z)$ là hiệu của vế phải và vế trái, từ bổ đề

\[(y^2+1)(z^2+1) \leqslant \left[1+\left(\frac{y+z}{2}\right)^2 \right]^2,\]

ta chứng minh được $f(x,y,z) \geqslant f(x,t,t).$

 

Bây giờ ta sẽ chứng minh $f(x,t,t) \geqslant 0,$ bất đẳng thức này tương đương với

\[1+{{\left( \frac{x+2t}{3} \right)}^{2}}-\sqrt[3]{({{x}^{2}}+1){{({{t}^{2}}+1)}^{2}}}\ge 0,\]

hay là

$${{\left[ {{(x+2t)}^{2}}+9 \right]}^{3}}-729({{x}^{2}}+1){{({{t}^{2}}+1)}^{2}}\ge 0,$$

hoặc

\[\underbrace{\big[{{x}^{4}}+14{{x}^{3}}t+(87{{t}^{2}}+27){{x}^{2}}+(320{{t}^{3}}+270t)x+64{{t}^{4}}-297{{t}^{2}}-486\big]}_{P}(x-t)^2 \geqslant 0.\]

Chú ý rằng

\[\begin{aligned}P=&\left[320t^3+(87x+87)t^2+(14x^2+14x)t+x^3+x^2+28x+312\right] (x-1)\\&+(64t^3+384t^2+174t+284x+174)(t-1) \geqslant 0.\end{aligned}\]

Nên ta có điều phải chứng minh.




#691260 $\frac{a+b+c}{3}\geq \sqrt[5]{\frac{a^{2}+b^{2}+c^{2...

Đã gửi bởi Nguyenhuyen_AG on 22-08-2017 - 00:02 trong Bất đẳng thức và cực trị

Cho a,b,c>0 thỏa abc=1.CMR: $\frac{a+b+c}{3}\geq \sqrt[5]{\frac{a^{2}+b^{2}+c^{2}}{3}}$

 

Bất đẳng thức cần chứng minh tương đương với

\[f=(a+b+c)^5 - 81(a^2+b^2+c^2) \geqslant 0.\]

Ta có

\[f = \frac12 \sum (a^3+b^3+21c^3+2a^2b+2ab^2)(a-b)^2+5\sum c(a-b)^4 \geqslant 0.\]




#691259 Chứng minh $(a^2 +2)(b^2+2)(c^2+2) \ge 3(a+b+c)^2$

Đã gửi bởi Nguyenhuyen_AG on 21-08-2017 - 23:47 trong Bất đẳng thức và cực trị

Anh ơi cho em hỏi là anh dùng kĩ thuật gì để ra cái này ạ

 

Anh dùng hệ số bất định.




#691236 BẤT ĐẲNG THỨC

Đã gửi bởi Nguyenhuyen_AG on 21-08-2017 - 20:49 trong Bất đẳng thức và cực trị

Cho a,b,c > 0. Chứng minh rằng:

                                             $a^{3}+b^{3}+c^{3}\geq ab^{2}+bc^{2}+ca^{2}$

 

Giả sử $c=\min\{a,b,c\}$ khi đó

\[a^{3}+b^{3}+c^{3}-(ab^{2}+bc^{2}+ca^{2}) = (a+b)(a-b)^2+(c+a)(a-c)(b-c) \geqslant 0.\]




#691166 Chứng minh $(a^2 +2)(b^2+2)(c^2+2) \ge 3(a+b+c)^2$

Đã gửi bởi Nguyenhuyen_AG on 20-08-2017 - 20:59 trong Bất đẳng thức và cực trị

Cho $a,b,c >0$. Chứng minh:

$(a^2 +2)(b^2+2)(c^2+2) \ge 3(a+b+c)^2$

 

Ta có

\[\text{Vế trái  -  Vế phải} = \frac{\displaystyle 3\sum (c^2+5)(ab-1)^2 + \sum (a+b-2c)^2 + 3\left(\sum ab -3\right)^2}{9}.\]




#690911 Chứng minh $\sum {\frac{{a{b^2}}...

Đã gửi bởi Nguyenhuyen_AG on 18-08-2017 - 18:44 trong Bất đẳng thức và cực trị

Có một cách đơn giản hớn là chứng minh

\[\sum \frac{x}{x^3+1} \leqslant \frac34 \sum \frac{x+1}{x^2+x+1} \leqslant \frac32.\]




#690707 xab+ybc+zca

Đã gửi bởi Nguyenhuyen_AG on 16-08-2017 - 22:41 trong Bất đẳng thức và cực trị

có ai biết cách làm dạng này ko?

a)Cho a+b+c=1. Tìm max A= xab+ ybc +zca (x,y,z là các stn)

b ) Cho ab+ bc+ca=1 .tìm min B= xa^2+ yb^2 +zc^2 (x,y,z là các stn)

Tks nh :))

Câu a dùng tam thức, câu b dùng Cauchy-Schwarz.




#690419 $\sum \frac{a}{a+b}+\frac{abc...

Đã gửi bởi Nguyenhuyen_AG on 13-08-2017 - 13:23 trong Bất đẳng thức và cực trị

Còn link không anh ? 

 

Link trên VMF anh thử tìm nhưng không ra: http://k2pi.net.vn/s...ead.php?p=78215




#690418 $1+\frac{1}{x^2}$ ≥ $cosA+x(cosB+cosC...

Đã gửi bởi Nguyenhuyen_AG on 13-08-2017 - 13:05 trong Hình học phẳng

 

Cho tam giác ABC là một tam giác bất kì. Chứng minh rằng với mọi số x ta đều có

 

$1+\frac{1}{2}x^2$ ≥ $cosA+x(cosB+cosC)$

 

Xem hiệu hai vế là một tam thức bậc hai theo $x$ với hệ số của $x^2$ dương. Để chứng minh tam thức này không âm ta chỉ cần chứng minh biệt thức $\Delta$ của nó luôn $\leqslant 0$ là được.

 

Thật vậy

\[\begin{aligned}\Delta & = (\cos B + \cos C)^2 + 2\cos A - 2 \\&= \frac14\left(\frac{c^2+a^2-b^2}{ca}+\frac{a^2+b^2-c^2}{ab}\right)^2 + \frac{b^2+c^2-a^2}{bc}-2 \\&= -\frac{(b-c)^2\left[2(a^2b^2+b^2c^2+c^2a^2)-a^4-b^4-c^4\right] }{4a^2b^2c^2} \leqslant 0.\end{aligned}\]

Ta có điều phải chứng minh.